Why is the statement in the question stem not a...
From the evidence of whiskers which I understand is a premise, it seems like they conclude that t...
Andrew on October 19, 2022
  • June 2014 LSAT
  • SEC3
  • Q15
1
Reply
Reason for eliminating D
Hi I read the post concerning the elimination of answer-choice D. Nevertheless, I still hav...
Mazen on May 27, 2022
  • June 2014 LSAT
  • SEC3
  • Q23
5
Replies
PT 72, S3, Q11
I don't really understand why this is A. Can you please explain? Thank you!
Sean on December 28, 2020
  • June 2014 LSAT
  • SEC3
  • Q11
8
Replies
Why not B?
Could you please explain. Thanks.
on October 22, 2020
  • June 2014 LSAT
  • SEC3
  • Q7
1
Reply
Why not D?
There have been a lot of questions as to why D doesn't work in the last thread. Can someone pleas...
on October 18, 2020
  • June 2014 LSAT
  • SEC3
  • Q11
1
Reply
C vs D
I was torn between C and D and ended up going with D. Can someone please explain this question? T...
on August 11, 2020
  • June 2014 LSAT
  • SEC3
  • Q23
4
Replies
Why is E correct? Why is B incorrect?
Thanks
Ryan on August 11, 2020
  • June 2014 LSAT
  • SEC3
  • Q21
4
Replies
June 2014 Practice Test, Logical Reasoning #13
Can someone explain all the answer choices, I chose A. I didn't think that any of the principles ...
Sheikh on July 7, 2020
  • June 2014 LSAT
  • SEC3
  • Q13
1
Reply
June 2014 Practice Test, Logical Reasoning #16
Why is B incorrect?
Sheikh on July 7, 2020
  • June 2014 LSAT
  • SEC3
  • Q16
1
Reply
June 2014 Practice Test, Logical Reasoning #17
Why is A wrong?
Sheikh on July 7, 2020
  • June 2014 LSAT
  • SEC3
  • Q17
1
Reply
B vs. D
I struggled a lot with this question. The issue I had with the wording of B was that amusia doesn...
Avi on June 24, 2020
  • June 2014 LSAT
  • SEC3
  • Q20
3
Replies
Errors in Reasoning
Hi, I'm having a hard time with this problem and seeing why 'A' would be right. Thanks!
Evan on May 4, 2020
  • June 2014 LSAT
  • SEC3
  • Q7
8
Replies
Why is C correct? Why is B incorrect?
Thanks
Ryan on October 12, 2019
  • June 2014 LSAT
  • SEC3
  • Q6
1
Reply
Why is D correct? Why is E incorrect?
Thanks
Ryan on October 12, 2019
  • June 2014 LSAT
  • SEC3
  • Q10
1
Reply
Why is D correct? Why is E incorrect?
Thanks
Ryan on October 12, 2019
  • June 2014 LSAT
  • SEC3
  • Q25
1
Reply
Help please!
I am confused on how C is the correct answer. Any help is appreciated!
on June 1, 2019
  • June 2014 LSAT
  • SEC3
  • Q18
2
Replies
Can someone explain the differences among b,c,a...
Any explanation would be great. Thank you!
HAMEE on June 1, 2019
  • June 2014 LSAT
  • SEC3
  • Q20
1
Reply
Question
Why is A correct?
Claire on May 23, 2019
  • June 2014 LSAT
  • SEC3
  • Q8
1
Reply
Confused
How is A the correct answer?
on February 26, 2019
  • June 2014 LSAT
  • SEC3
  • Q24
2
Replies
Between A and B
Is the conclusion in this argument "Thrinaxodon was probably warm blooded" or "Thrinaxadon was pr...
on December 27, 2018
  • June 2014 LSAT
  • SEC3
  • Q15
1
Reply